Cryptotastic

Algebra Level 2

A B C D + E F G H \begin{array} { l l l } & A & B \\ & C & D \\ + & E & F \\ \hline & G & H \\ \end{array}

In the above cryptogram, all the letters represent distinct digits from 1 to 9.

What is the minimum possible value of G H \overline{GH} ?


Inspiration .

For a greater challenge, try Cryptotastic #2 .


The answer is 78.

This section requires Javascript.
You are seeing this because something didn't load right. We suggest you, (a) try refreshing the page, (b) enabling javascript if it is disabled on your browser and, finally, (c) loading the non-javascript version of this page . We're sorry about the hassle.

9 solutions

Venkatachalam J
May 2, 2017

how about 1 5 , 3 6, 4 7, 9 8. This gives 98 as the maximum possible, all are distinct numbers

Srini Mandalapu - 4 years, 1 month ago

Log in to reply

Great. Can you elaborate how you find the maximum possible value for the current problem.

Venkatachalam J - 4 years, 1 month ago

Log in to reply

Step 1: Start GH with 98 (which is the maximum possible combination) Step 2: Fill B, D and F with next highest possible numbers 5, 6 and 7 Step 3: First try A, C and E with next highest possible numbers – 2, 3 and 4 , this gives a total of 108. Replace 2 with 1 results a total of 98. So the initial pick of 98 for GH worked. 1 5 3 6 4 7 9 8

Srini Mandalapu - 4 years, 1 month ago

Log in to reply

@Srini Mandalapu Yes. I like your solution approach to find the "maximum possible value" for the current problem.

Venkatachalam J - 4 years, 1 month ago

This greedy approach doesn't prove that it's in fact the smallest possible, because it's possible that some other approach will give a smaller sum.

Ivan Koswara - 4 years, 1 month ago

you should have put in 'greater than zero'.

Michael Lewis - 4 years, 1 month ago

If we can, we will want A , C , E A,C,E to be 1 , 2 , 3 1,2,3 in some order, so without loss of generality let ( A , C , E ) = ( 1 , 2 , 3 ) (A,C,E) = (1,2,3) . Now of the digits remaining, the minimum sum of any three of them will be 4 + 5 + 6 = 15 4 + 5 + 6 = 15 , so the best we can do is have G = 7 G = 7 . Now for ( B , D , F ) (B,D,F) we can't use ( 4 , 5 , 6 ) (4,5,6) since that will make H = 5 H = 5 . The next best options would be ( 4 , 5 , 8 ) (4,5,8) , but that would make H = 7 H = 7 , which is already taken. We move on to ( 4 , 5 , 9 ) (4,5,9) which will make H = 8 H = 8 and all the digits distinct. So the minimum desired value is G H = 78 \overline{GH} = \boxed{78} .

(Note that ( B , D , F ) = ( 4 , 6 , 8 ) (B,D,F) = (4,6,8) would also yield G H = 78 \overline{GH} = 78 but this would involve a repeated digit.)

Edit: To be complete, the only other choice for ( A , C , E ) (A,C,E) to have a minimum less than 80 80 would be if we had ( A , C , E ) = ( 1 , 2 , 4 ) (A,C,E) = (1,2,4) , but as the least digits remaining for B , D , F B,D,F are 3 , 5 , 6 3,5,6 the final sum would be necessarily greater than 80 80 .

Nice! If anyone thought 10+24+35, like me, note that that cannot work because the problem states

"all the letters represent distinct digits from 1 to 9."

which does not include zero. The note is a bit unnecessary, then.

Alex Li - 4 years, 1 month ago

Log in to reply

Thanks! You're right about the note being unnecessary; someone else added it, apparently, so I've just removed it.

Brian Charlesworth - 4 years, 1 month ago

Nice one Brian

Sharath Achiever - 4 years, 1 month ago

Log in to reply

Thanks! :)

Brian Charlesworth - 4 years, 1 month ago

17+36+45=98 how is this wrong?

Shiah Lints - 4 years, 1 month ago

Log in to reply

Sorry I was looking for the max

Shiah Lints - 4 years, 1 month ago

18 + 25 + 36 = 79 is also less than 80 but is admittedly greater than 78

Ian Newman - 4 years, 1 month ago

Log in to reply

Yes, quite right. There are a total of 6 solutions, but 78 is the minimum, (by a whisker, as your sum reveals).

Brian Charlesworth - 4 years, 1 month ago

Log in to reply

Thank you - close, but no cigar for me!

Ian Newman - 4 years, 1 month ago

16+34+25= 75

Shujath Hussain - 4 years, 1 month ago

Log in to reply

The digits must all be distinct. Your solution uses 5 twice so is not valid.

Brian Charlesworth - 4 years, 1 month ago

I guess the question is not worded properly. how could 78 be the minimum when 75 comes about with legitimate use of the numbers?

Vahe Apelian - 4 years, 1 month ago

Log in to reply

How did you get 75? Others have mentioned 16 + 34 + 25 = 75 as a solution, but the digit 5 is used twice in this case.

Brian Charlesworth - 4 years, 1 month ago

For all digits to be distinct I only got one solution: 14 + 39 + 25 = 78. The order of the three numbers are not taken into account; I ignored the associative and commutative laws.

Anelize Theron van Biljon - 4 years, 1 month ago

Log in to reply

There are other solutions. Here's one: 15+26+38=79.

Ivan Koswara - 4 years, 1 month ago

what if B= 3, D=4, F=2? H would be 9. Also, A=1, C=2, and E=5? That would make G=8, Therefore the answer would be 89.But you can also make it higher (98)

Foxy The Pro - 4 years, 1 month ago

Log in to reply

In your example F and C would both be 2, but the question requires them to be distinct. Also, the question asks for the minimum possible value, not the maximum.

Brian Charlesworth - 4 years, 1 month ago

Log in to reply

thank you!

Foxy The Pro - 4 years, 1 month ago

I think it's wrong because we can do this: 16 25 34 And the answer will be 75

Elias Karayanni - 4 years, 1 month ago

Log in to reply

The digits must all be distinct. Your example uses the digit 5 twice, so it is not a valid solution.

Brian Charlesworth - 4 years, 1 month ago

This doesn't prove that it's the minimum possible, since you haven't checked the cases where { A , C , E } { 1 , 2 , 3 } \{A,C,E\} \neq \{1,2,3\} and such.

Ivan Koswara - 4 years, 1 month ago

Log in to reply

True, we could have ( A , C , E ) = ( 1 , 2 , 4 ) (A,C,E) = (1,2,4) . I've added an edit to my solution to deal with this case.

Brian Charlesworth - 4 years, 1 month ago

Log in to reply

Yes, that works. 78 is achievable, so the sum should be less than 80; that means A + C + E 7 A+C+E \le 7 , so those two possibilities suffice.

Ivan Koswara - 4 years, 1 month ago

Log in to reply

@Ivan Koswara The only possible solution for (A,B,C) is (1,2,3) when we try to find the minimum value.

When we use (1,2,3), we yield G = 7 since (4,5,6) > 10.

However, if we use (1,2,4), we yield G = 8 since (3,5,6) > 10.

Therefore, 7 < 8 and (1,2,3) is required for this solution.

Wesley McCormick - 4 years, 1 month ago

Log in to reply

@Wesley McCormick Exactly. But you still need to write that result out, to explain that A + C + E 7 A+C+E \ge 7 can be immediately ruled out; otherwise the solution is incomplete.

Ivan Koswara - 4 years, 1 month ago

16+25+34 = 75 and this is the smallest

Darius Pypkoskey - 4 years, 1 month ago

Log in to reply

The digits must all be distinct. You have used the digit 5 twice in your solution.

Brian Charlesworth - 4 years, 1 month ago

5 is used twice there, and it states for numbers 1 to 9 to be used once.

Foxy The Pro - 4 years, 1 month ago
Matthew Feig
May 1, 2017

Even without trying for the minimum value, I was surprised how few ways there are to fill in the digits at all. I only came up with five possible values of GH.

In keeping the sum under 100, it is hard to avoid repeating a digit.

You can give the follow-up question "Cryptotastic #2" a try as well, which asks for the sum of all possible values. You have found five, but you'll need to look for a sixth before adding them up.

Brian Charlesworth - 4 years, 1 month ago

If one uses 0, then the answer can be 69 ( = 10 + 24 + 35)

Dyna Sukas - 4 years, 1 month ago

Oops. I meant to say six possible values.

Matthew Feig - 4 years, 1 month ago

There is no requirement against using zero, so:

10 + 24 + 35 = 69.

Keith Edgette - 4 years, 1 month ago

Log in to reply

Well, I see it now. Facebook doesn't list the requirement, but this page does. My mistake.

Keith Edgette - 4 years, 1 month ago

Log in to reply

Thanks for mentioning that the Facebook post has omitted this requirement.

@Calvin Lin Would it possible to edit the Facebook post of this question to make it clear that only digits 1 through 9 can be used? Thanks. :)

Brian Charlesworth - 4 years, 1 month ago

Log in to reply

@Brian Charlesworth Sorry about that. I've updated the FB post

Calvin Lin Staff - 4 years, 1 month ago

Log in to reply

@Calvin Lin Great! Thanks for doing that so quickly. :)

Brian Charlesworth - 4 years, 1 month ago

15 + 36 + 47 = 98

Carl Strohmenger - 4 years, 1 month ago

Log in to reply

Yes, that is the maximum possible value, but the question asks for the minimum possible value.

Brian Charlesworth - 4 years, 1 month ago

14+25+36 = 75

CJ L LeBlanc - 4 years, 1 month ago

Log in to reply

:O was wrong 5 ...

CJ L LeBlanc - 4 years, 1 month ago
Krishna Deb
May 2, 2017

To keep the number GH restricted to 2 digits, that is, to arrive at any solution that even satisfies the question is 70% of the work done.

Obviously, we'll want A, C, E, G to be as small as possible.

So we'll assume A, C, E to be 1, 2 and 3. Now we only have larger numbers left.

We'll first asume B, D ,F to be 4, 5 and 6, as that is the logical conclusion.

However, on adding the resultant digits, it appears that H turns out to be 5! We can't afford a repetition!

So we keep increasing the value of F, from 5 to 6 and beyond, while keeping B and D at the lowest possible values.

Finally we get something like this - 14 + 25 + 39 = 78 No repetition.

And using the smallest possible values.

P.S. - There are actually really few ways to solve this problem. If you have a good one, share it!!

Jesse Nieminen
May 4, 2017

Let's first try to find a lower bound for the value of G H \overline{GH} .

Clearly, 1 + 2 + 3 = 6 A + C + E G 1+2+3=6\leq A+C+E\leq G

However, G 6 G\neq6 because if A + C + E = 6 A+C+E=6 , then B + D + F > 4 + 5 + 6 > 9 B+D+F>4+5+6>9 forcing G > 6 G>6 .

Now we check by trial and error which (if any) values of H H solve the cryptogram when G = 7 G=7 and take the smallest.

If A + C + E = 7 A+C+E=7 , then B + D + F > 9 B+D+F>9 forcing G > 7 G>7 , hence A + C + E = 6 A+C+E=6 . ( 1 , 2 , 3 , 7 1,2,3,7 are reserved.)

If H < 7 H<7 , we have B + D + F 4 + 5 + 8 > 16 B+D+F\geq 4+5+8>16 which either carries at least 2 2 over to G G or forces H > 6 H>6 .

If H = 7 H=7 , we have G 7 G\neq7 .

If H = 8 H=8 we have ( A , B , C , D , E , F , G , H ) = ( 1 , 4 , 2 , 5 , 3 , 9 , 7 , 8 ) \left(A,B,C,D,E,F,G,H\right)=\left(1,4,2,5,3,9,7,8\right) as a solution.

Since G 7 G\geq7 and if G = 7 G=7 , H 8 H\geq8 and if H = 8 H=8 , G H 78 \overline{GH}\geq78 and since G H = 78 \overline{GH}=78 is possible, the minimum value of G H \overline{GH} is 78 \boxed{78} .

Alkis Piskas
May 4, 2017

I selected the minimum for A,C,E (=1,2,3). Then I selected the next minimum for B,D.F (=4,5,6). But F=6/7/8 doesn't work, so F=9. Sum: 14+25+39=78. However, this is a trial-and-error method and I was "lucky" to get to the solution fast! So, I would like to see a solution that is obtained directly based on an algorithm or other logic method, but w/o trying ...

Why doesnt 6 work for F?

Oana Alexandra - 3 years ago
Robert DeLisle
May 6, 2017

The answer is 78 demonstrated as follows:

Best try for a minimum value is A, B, C as 1,2,3 in order to minimize the ten's place.

The order of the digits in each column have no effect on the sum. This is evident if the sum is expressed as GH = 10 (A+C+E) + (B+D+F)). So let A, C, E be 1,2,3 in any order. (1)

Any choice of digits for the one's places will have a carry of at least one since the minimum choice {4,5,6} sums to 15 (and a maximum of 2 with {7,8,9}), Therefore the minimum value of G is 7 and our choices for B,D,E are limited to {4,5,6,8,9).

The minimum choices for B.D,F in ascending order are {4,5,6} with H=5 that fails by duplicating 5, {4,5,8} with H = 7 that fails by duplicating G = 7, and {4,5,9} with H = 8 that solves the puzzle with GH = 78. The solution is minimal because all minimum possibilities were exhausted in the process of finding the solution, none lower can exist.


(1) I hope you find that clear and concise with the additional virtue of avoiding the dreaded "assume without loss of generality".

Lowest total is 14+25+36=75 but this ends up repeating the digit 5 and we we can't have that as no two letters are the same numbers. So you just increase the 36 to 37,38,39 and you quickly discover that 39 will increase the total to 78, now leaving you with all different digits and by definition the lowest total. Hope that makes sense!

Alex Elie-Pierre
May 3, 2017

A=1; B=2; C=3; D=4; E=5; F=6; G=7; H=8

This is not right. Is the sum 12 + 34 + 56 equal to 78?

Pranshu Gaba - 4 years, 1 month ago

Log in to reply

Sorry... to be clear... I was saying that I simply numbered them as I illustrated and came up with the correct answer by what seems to be total luck!

Alex Elie-Pierre - 4 years ago

0 pending reports

×

Problem Loading...

Note Loading...

Set Loading...